Class 12 NCERT Maths – Chapter 3 (Matrices) – Exercise 3.2 Solutions

Question 1:  Let
A = \begin{bmatrix} 2 & 4 \\ 3 & 2 \end{bmatrix},\; B = \begin{bmatrix} 1 & 3 \\ -2 & 5 \end{bmatrix},\; C = \begin{bmatrix} -2 & 5 \\ 3 & 4 \end{bmatrix}

Find: 
(i) A + B (ii) A - B (iii) 3A - C (iv) AB (v) BA

Solution
(i) A + B 

A+B = \begin{bmatrix} 2+1 & 4+3 \\ 3+(-2) & 2+5 \end{bmatrix}= \begin{bmatrix} 3 & 7 \\ 1 & 7 \end{bmatrix}

(ii) A - B

A-B = \begin{bmatrix} 2-1 & 4-3 \\ 3-(-2) & 2-5 \end{bmatrix}= \begin{bmatrix} 1 & 1 \\ 5 & -3 \end{bmatrix}

(iii) 3A - C

3A = \begin{bmatrix} 6 & 12 \\ 9 & 6 \end{bmatrix} \\\\
3A - C = \begin{bmatrix} 6-(-2) & 12-5 \\ 9-3 & 6-4 \end{bmatrix}= \begin{bmatrix} 8 & 7 \\ 6 & 2 \end{bmatrix}

(iv) AB 

Row 1 × Col 1 : 2(1)+4(-2)=-6 
Row 1 × Col 2 : 2(3)+4(5)=26 
Row 2 × Col 1 : 3(1)+2(-2)=-1 
Row 2 × Col 2 : 3(3)+2(5)=19

AB = \begin{bmatrix}-6 & 26 \\ -1 & 19\end{bmatrix}

(v)  BA

Row 1 × Col 1 : 1(2)+3(3)=11 
Row 1 × Col 2 : 1(4)+3(2)=10 
Row 2 × Col 1 : -2(2)+5(3)=11 
Row 2 × Col 2 : -2(4)+5(2)=2

BA = \begin{bmatrix}11 & 10 \\ 11 & 2\end{bmatrix}

Question 2: Compute the following

(i) \begin{bmatrix} a & b \\ -b & a \end{bmatrix} + \begin{bmatrix} a & b \\ b & a \end{bmatrix}

(ii) \begin{bmatrix} a^2+b^2 & b^2+c^2 \\ a^2+c^2 & a^2+b^2 \end{bmatrix} + \begin{bmatrix} 2ab & 2bc \\ -2ac & -2ab \end{bmatrix}

(iii) \begin{bmatrix}-1 & 4 & -6\\8 & 5 & 16\\2 & 8 & 5\end{bmatrix} + \begin{bmatrix}12 & 7 & 6\\8 & 0 & 5\\3 & 2 & 4\end{bmatrix}

(iv) \begin{bmatrix}\cos^2x & \sin^2x\\ \sin^2x & \cos^2x\end{bmatrix} + \begin{bmatrix}\sin^2x & \cos^2x\\ \cos^2x & \sin^2x\end{bmatrix}

Solutions:

(i) \begin{bmatrix} a & b \\ -b & a \end{bmatrix} + \begin{bmatrix} a & b \\ b & a \end{bmatrix} =\begin{bmatrix} a+a & b+b \\ -b+b & a+a \end{bmatrix}=\begin{bmatrix} 2a & 2b \\ 0 & 2a \end{bmatrix}

(ii) \begin{bmatrix} a^2+b^2 & b^2+c^2 \\ a^2+c^2 & a^2+b^2 \end{bmatrix} + \begin{bmatrix} 2ab & 2bc \\ -2ac & -2ab \end{bmatrix}

Add entry‑wise:

\begin{aligned}&(1,1):\; a^2+b^2+2ab=(a+b)^2\\[2pt]&(1,2):\; b^2+c^2+2bc=(b+c)^2\\[2pt]&(2,1):\; a^2+c^2-2ac=(a-c)^2\\[2pt]&(2,2):\; a^2+b^2-2ab=(a-b)^2\end{aligned}

\begin{bmatrix}(a+b)^2 & (b+c)^2 \\ (a-c)^2 & (a-b)^2\end{bmatrix}

(iii) \begin{bmatrix}-1 & 4 & -6\\8 & 5 & 16\\2 & 8 & 5\end{bmatrix} + \begin{bmatrix}12 & 7 & 6\\8 & 0 & 5\\3 & 2 & 4\end{bmatrix}

=\begin{bmatrix} -1+12 & 4+7 & -6+6 \\ 8+8 & 5+0 & 16+5 \\ 2+3 & 8+2 & 5+4 \end{bmatrix}=\begin{bmatrix} 11 & 11 & 0 \\ 16 & 5 & 21 \\ 5 & 10 & 9 \end{bmatrix}

(iv) \begin{bmatrix}\cos^2x & \sin^2x\\ \sin^2x & \cos^2x\end{bmatrix} + \begin{bmatrix}\sin^2x & \cos^2x\\ \cos^2x & \sin^2x\end{bmatrix}

Use \sin^2x+\cos^2x=1:

=\begin{bmatrix}1 & 1\\1 & 1\end{bmatrix} = \text{each entry }=1

Question 3. Compute

(i) \begin{bmatrix} a & b \\ -b & a \end{bmatrix} \times \begin{bmatrix} a & -b \\ b & a \end{bmatrix}

Solution

Entry (1,1): a\cdot a + b\cdot b = a^2 + b^2 
Entry (1,2): a(-b) + b a = -ab + ab = 0 
Entry (2,1): -b\cdot a + a b = -ab + ab = 0 
Entry (2,2): -b(-b) + a a = b^2 + a^2 = a^2 + b^2

\begin{bmatrix} a^2+b^2 & 0 \\ 0 & a^2+b^2 \end{bmatrix}

(ii) \begin{bmatrix} 1 \\ 2 \\ 3 \end{bmatrix} \times \begin{bmatrix} 2 & 3 & 4 \end{bmatrix}

The first matrix is 3 × 1 (column) and the second is 1 × 3 (row), so the product is a 3 × 3 matrix obtained by multiplying each entry of the column with each entry of the row.

\begin{aligned}&\text{Row 1}:\; 1\times2=2,\;1\times3=3,\;1\times4=4\\[2pt]&\text{Row 2}:\; 2\times2=4,\;2\times3=6,\;2\times4=8\\[2pt]&\text{Row 3}:\; 3\times2=6,\;3\times3=9,\;3\times4=12\end{aligned}

\begin{bmatrix} 2 & 3 & 4 \\ 4 & 6 & 8 \\ 6 & 9 & 12 \end{bmatrix}

(iii). \begin{bmatrix} 1 & -2 \\ 2 & 3 \end{bmatrix} \times \begin{bmatrix} 1 & 2 & 3 \\ 2 & 3 & 1 \end{bmatrix}

\begin{aligned}(1,1):&\;1\cdot1+(-2)\cdot2=-3\\[2pt](1,2):&\;1\cdot2+(-2)\cdot3=-4\\[2pt](1,3):&\;1\cdot3+(-2)\cdot1=1\\[4pt] (2,1):&\;2\cdot1+3\cdot2=8\\[2pt](2,2):&\;2\cdot2+3\cdot3=13\\[2pt](2,3):&\;2\cdot3+3\cdot1=9\end{aligned}

\begin{bmatrix} -3 & -4 & 1 \\ 8 & 13 & 9 \end{bmatrix}

(iv). \begin{bmatrix} 2 & 3 & 4 \\ 3 & 4 & 5 \\ 4 & 5 & 6 \end{bmatrix} \times \begin{bmatrix} 1 & -3 & 5 \\ 0 & 2 & 4 \\ 3 & 0 & 5 \end{bmatrix}

Solution: Compute row 1 as illustration; the rest follow similarly.

(1,1): 2\cdot1+3\cdot0+4\cdot3=14\qquad \\\\ (1,2): 2(-3)+3\cdot2+4\cdot0=0\qquad \\\\(1,3): 2\cdot5+3\cdot4+4\cdot5=42

Proceeding for rows 2 and 3:

\begin{bmatrix} 14 & 0 & 42 \\ 18 & -1 & 56 \\ 22 & -2 & 70 \end{bmatrix}

(v). \begin{bmatrix} 2 & 1 \\ 3 & 2 \\ -1 & 1 \end{bmatrix} \times \begin{bmatrix} 1 & 0 & 1 \\ -1 & 2 & 1 \end{bmatrix}

Solution

\begin{aligned}(1,1):&\;2\cdot1+1(-1)=1 & (1,2):&\;2\cdot0+1\cdot2=2 & (1,3):&\;2\cdot1+1\cdot1=3\\[2pt](2,1):&\;3\cdot1+2(-1)=1 & (2,2):&\;3\cdot0+2\cdot2=4 & (2,3):&\;3\cdot1+2\cdot1=5\\[2pt](3,1):&\;-1\cdot1+1(-1)=-2 & (3,2):&\;-1\cdot0+1\cdot2=2 & (3,3):&\;-1\cdot1+1\cdot1=0\end{aligned}

\begin{bmatrix} 1 & 2 & 3 \\ 1 & 4 & 5 \\ -2 & 2 & 0 \end{bmatrix}

(vi) \begin{bmatrix} 3 & -1 & 3 \\ -1 & 0 & 2 \end{bmatrix} \times \begin{bmatrix} 2 & -3 \\ 1 & 0 \\ 3 & 1 \end{bmatrix}

Solution

(1,1): 3\cdot2 + (-1)\cdot1 + 3\cdot3 = 14 \quad \\\\ (1,2): 3(-3)+(-1)\cdot0+3\cdot1 = -6

(2,1): (-1)\cdot2 + 0\cdot1 + 2\cdot3 = 4 \quad \\\\ (2,2): (-1)(-3)+0\cdot0+2\cdot1 = 5

\begin{bmatrix} 14 & -6 \\ 4 & 5 \end{bmatrix}
Question 4.

If A = \begin{bmatrix} 1 & 2 & -3 \\ 5 & 0 & 2 \\ 1 & -1 & 1 \end{bmatrix},\;B = \begin{bmatrix} 3 & -1 & 2 \\ 4 & 2 & 5 \\ 2 & 0 & 3 \end{bmatrix},\;C = \begin{bmatrix} 4 & 1 & 2 \\ 0 & 3 & 2 \\ 1 & -2 & 3 \end{bmatrix}

Find (i) A+B, (ii) B-C and verify that A + (B - C) = (A + B) - C.

Solution

(i) Addition
A+B = \begin{bmatrix} 1+3 & 2 + (-1) & -3 + 2 \\ 5+4 & 0+2 & 2+5 \\ 1+2 & -1+0 & 1+3 \end{bmatrix}= \begin{bmatrix} 4 & 1 & -1 \\ 9 & 2 & 7 \\ 3 & -1 & 4 \end{bmatrix}

(ii) Subtraction
B-C = \begin{bmatrix} 3-4 & -1-1 & 2-2 \\ 4-0 & 2-3 & 5-2 \\ 2-1 & 0-(-2) & 3-3 \end{bmatrix}= \begin{bmatrix} -1 & -2 & 0 \\ 4 & -1 & 3 \\ 1 & 2 & 0 \end{bmatrix}

(iii) Verification
A + (B - C) = \begin{bmatrix} 0 & 0 & -3 \\ 9 & -1 & 5 \\ 2 & 1 & 1 \end{bmatrix}, \quad(A + B) - C = \begin{bmatrix} 0 & 0 & -3 \\ 9 & -1 & 5 \\ 2 & 1 & 1 \end{bmatrix}

Since the two matrices are equal, the property is verified.

Question 5.

If A = \begin{bmatrix} \tfrac23 & 1 & \tfrac53 \\\\ \tfrac13 & \tfrac23 & \tfrac43 \\\\ \tfrac73 & 2 & \tfrac23 \end{bmatrix},\;B = \begin{bmatrix} \tfrac25 & \tfrac35 & 1 \\\\ \tfrac15 & \tfrac25 & \tfrac45 \\\\ \tfrac75 & \tfrac65 & \tfrac25 \end{bmatrix},

find 3A - 5B.

Solution
3A = \begin{bmatrix} 2 & 3 & 5 \\ 1 & 2 & 4 \\ 7 & 6 & 2 \end{bmatrix}, \quad 5B = \begin{bmatrix} 2 & 3 & 5 \\ 1 & 2 & 4 \\ 7 & 6 & 2 \end{bmatrix}

Clearly 3A = 5B; therefore 3A - 5B = O_{3 \times 3}, the 3 × 3 zero matrix.

Question 6 Simplify

\cos\theta\,\begin{bmatrix}\cos\theta & \sin\theta\\-\sin\theta & \cos\theta\end{bmatrix} + \sin\theta\,\begin{bmatrix}\sin\theta & -\cos\theta\\ \cos\theta & \sin\theta\end{bmatrix}.

Solution

\cos\theta \begin{bmatrix}\cos\theta & \sin\theta\\ -\sin\theta & \cos\theta\end{bmatrix} = \begin{bmatrix}\cos^{2}\theta & \cos\theta\sin\theta\\ -\cos\theta\sin\theta & \cos^{2}\theta\end{bmatrix}

\sin\theta \begin{bmatrix}\sin\theta & -\cos\theta\\ \cos\theta & \sin\theta\end{bmatrix} = \begin{bmatrix}\sin^{2}\theta & -\sin\theta\cos\theta\\ \sin\theta\cos\theta & \sin^{2}\theta\end{bmatrix}

\text{Sum} = \begin{bmatrix}\cos^{2}\theta + \sin^{2}\theta & 0 \\ 0 & \cos^{2}\theta + \sin^{2}\theta\end{bmatrix} = \begin{bmatrix} 1 & 0 \\ 0 & 1 \end{bmatrix}

Question 7: Find x and y if

(i) X+Y=\begin{bmatrix}7&0\\2&5\end{bmatrix},\;X-Y=\begin{bmatrix}3&0\\0&3\end{bmatrix}

Solution (i)
Add the two equations to eliminate Y:

(X+Y)+(X-Y)=2X=\begin{bmatrix}10&0\\2&8\end{bmatrix}

\Rightarrow X=\tfrac12\begin{bmatrix}10&0\\2&8\end{bmatrix}=\begin{bmatrix}5&0\\1&4\end{bmatrix}

Substitute X into the first equation:

Y=\begin{bmatrix}7&0\\2&5\end{bmatrix}-\begin{bmatrix}5&0\\1&4\end{bmatrix}=\begin{bmatrix}2&0\\1&1\end{bmatrix}

(ii) 2X+3Y=\begin{bmatrix}2&3\\4&0\end{bmatrix},\;3X+2Y=\begin{bmatrix}2&-2\\-1&5\end{bmatrix}

Solution (ii)
Treat the two matrix equations like simultaneous equations.
Solve for X and Y using the coefficients:

Multiply the first equation by 3 and the second by 2 to align X:

\begin{aligned}6X+9Y&=\begin{bmatrix}6&9\\12&0\end{bmatrix}\\[4pt]6X+4Y&=\begin{bmatrix}4&-4\\-2&10\end{bmatrix}\end{aligned}

Subtract second equation from first:

5Y=\begin{bmatrix}2&13\\14&-10\end{bmatrix}\;\Rightarrow\;Y=\begin{bmatrix}\tfrac25&\tfrac{13}{5}\\\tfrac{14}{5}&-2\end{bmatrix}

Now substitute Y back into equation

2X=\begin{bmatrix}2&3\\4&0\end{bmatrix}-3\begin{bmatrix}\tfrac25&\tfrac{13}{5}\\\\\tfrac{14}{5}&-2\end{bmatrix}=\begin{bmatrix}2-\tfrac65&3-\tfrac{39}{5}\\\\4-\tfrac{42}{5}&0+6\end{bmatrix}=\begin{bmatrix}\tfrac{4}{5}&-\tfrac{24}{5}\\\\\tfrac{-2}{5}&6\end{bmatrix}

X=\begin{bmatrix}\tfrac{2}{5}&-\tfrac{12}{5}\\\\-\tfrac{1}{5}&3\end{bmatrix}

Question 8: Find X if Y=\begin{bmatrix}3&2\\1&4\end{bmatrix},\;2X+Y=\begin{bmatrix}1&0\\-3&2\end{bmatrix}

Solution:
Isolate X:
2X=\begin{bmatrix}1&0\\-3&2\end{bmatrix}-\begin{bmatrix}3&2\\1&4\end{bmatrix}

2X=\begin{bmatrix}-2&-2\\-4&-2\end{bmatrix}

X=\begin{bmatrix}-1&-1\\-2&-1\end{bmatrix}

Question 9: 2\begin{bmatrix}1&3\\0&x\end{bmatrix}+\begin{bmatrix}y&0\\1&2\end{bmatrix}=\begin{bmatrix}5&6\\1&8\end{bmatrix}

Solution:
2\begin{bmatrix}1&3\\0&x\end{bmatrix}=\begin{bmatrix}2&6\\0&2x\end{bmatrix}

Computing left hand side
\begin{aligned}2\begin{bmatrix}1&3\\0&x\end{bmatrix}+\begin{bmatrix}y&0\\1&2\end{bmatrix}&=\begin{bmatrix}2+y & 6+0\\ 0+1 & 2x+2\end{bmatrix}=\begin{bmatrix}2+y & 6\\1 & 2x+2\end{bmatrix}\end{aligned}

Equate with the given right‑hand matrix:

\begin{bmatrix}2+y & 6\\1 & 2x+2\end{bmatrix} = \begin{bmatrix}5&6\\1&8\end{bmatrix}

\begin{cases}2+y=5\\6=6\\1=1\\2x+2=8\end{cases}\;\Rightarrow\; y=3,\; x=3

\boxed{x=3,\;y=3}

Question 10: Solve for x, y, z, t if

2\begin{bmatrix}x & z\\ y & t\end{bmatrix}+3\begin{bmatrix}1 & -1\\ 0 & 2\end{bmatrix}=3\begin{bmatrix}3 & 5\\ 4 & 6\end{bmatrix}

Solution.

2\begin{bmatrix}x & z\\ y & t\end{bmatrix}=\begin{bmatrix}2x & 2z\\ 2y & 2t\end{bmatrix}

3\begin{bmatrix}1 & -1\\ 0 & 2\end{bmatrix}=\begin{bmatrix}3 & -3\\ 0 & 6\end{bmatrix}

\text{LHS}=\begin{bmatrix}2x+3 & 2z-3\\ 2y & 2t+6\end{bmatrix},\;\text{RHS}=\begin{bmatrix}9 & 15\\ 12 & 18\end{bmatrix}

\Rightarrow\begin{cases}2x+3=9\\2z-3=15\\2y=12\\2t+6=18\end{cases}\Rightarrow\boxed{x=3,\;y=6,\;z=9,\;t=6}

Question 11: If x\begin{bmatrix}2\\3\end{bmatrix}+y\begin{bmatrix}-1\\1\end{bmatrix}=\begin{bmatrix}10\\5\end{bmatrix}, find x and y.

Solution.

\begin{bmatrix}2x-y\\3x+y\end{bmatrix}=\begin{bmatrix}10\\5\end{bmatrix}\;\Rightarrow\;\begin{cases}2x-y=10\\3x+y=5\end{cases}

Adding both the equations:
5x=15\Rightarrow x=3;
substituting: y=-4.

\boxed{x=3,\;y=-4}

Question 12: Given

3\begin{bmatrix}x & y\\ z & w\end{bmatrix}=\begin{bmatrix}x & 6\\ -1 & 2w\end{bmatrix}+\begin{bmatrix}4 & x+y\\ z+w & 3\end{bmatrix}, find x, y, z, w.

Solution

\text{LHS}=\begin{bmatrix}3x & 3y\\3z & 3w\end{bmatrix}

\text{RHS}=\begin{bmatrix}x+4 & 6+x+y\\ z+w-1 & 2w+3\end{bmatrix}

Equating entries gives:

\begin{cases}3x=x+4\\3y=6+x+y\\3z=z+w-1\\3w=2w+3\end{cases}\;\Rightarrow\;\boxed{x=2,\;y=4,\;z=1,\;w=3}

Question 13: If

F(x)=\begin{bmatrix}\cos x & -\sin x & 0\\ \sin x & \cos x & 0\\ 0 & 0 & 1\end{bmatrix}

show that F(x)\,F(y)=F(x+y).

Solution:

Write the two matrices

F(x)=\begin{bmatrix}\cos x & -\sin x & 0\\ \sin x & \cos x & 0\\ 0 & 0 & 1\end{bmatrix},\;F(y)=\begin{bmatrix}\cos y & -\sin y & 0\\ \sin y & \cos y & 0\\ 0 & 0 & 1\end{bmatrix}

Multiply row by column** (only the top‑left 2×2 block needs work; the last row and column behave like a 1).

\begin{aligned}F(x)F(y)&=\begin{bmatrix}\cos x\cos y-\sin x\sin y & -\cos x\sin y-\sin x\cos y & 0\\[4pt]\sin x\cos y+\cos x\sin y & -\sin x\sin y+\cos x\cos y & 0\\[4pt]0 & 0 & 1\end{bmatrix}\end{aligned} 

Use angle‑addition identities**

\cos x\cos y-\sin x\sin y=\cos(x+y),\\\\ \sin x\cos y+\cos x\sin y=\sin(x+y)

\Rightarrow F(x)F(y)=\begin{bmatrix}\cos(x+y) & -\sin(x+y) & 0\\ \sin(x+y) & \cos(x+y) & 0\\ 0 & 0 & 1\end{bmatrix}=F(x+y)

Hence proved ✅.

Question 14: Show that

(a). \begin{bmatrix} 5&-1\\6 & 7 \end{bmatrix} \begin{bmatrix} 2 & 1 \\ 3 & 4 \end{bmatrix} \neq \begin{bmatrix} 2 & 1 \\ 3 & 4 \end{bmatrix} \begin{bmatrix} 5 & -1 \\ 6 & 7 \end{bmatrix}

(b). \begin{bmatrix} 1 & 2 & 3 \\ 0 & 1 & 0 \\ 1 & 1 & 0 \end{bmatrix} \begin{bmatrix} -1 & 1 & 0 \\ \phantom{-}0 & -1 & 1 \\ \phantom{-}2 & 3 & 4 \end{bmatrix} \neq \begin{bmatrix} -1 & 1 & 0 \\ \phantom{-}0 & -1 & 1 \\ \phantom{-}2 & 3 & 4 \end{bmatrix} \begin{bmatrix} 1 & 2 & 3 \\ 0 & 1 & 0 \\ 1 & 1 & 0 \end{bmatrix}

Solution (a):

A=\begin{bmatrix}5&-1\\6&7\end{bmatrix},\;B=\begin{bmatrix}2&1\\3&4\end{bmatrix}

Compute AB.

AB=\begin{bmatrix}5\times2+(-1)\times3 & 5\times1+(-1)\times4\\ 6\times2+7\times3 & 6\times1+7\times4\end{bmatrix}=\begin{bmatrix}7&1\\33&34\end{bmatrix}

Step 2 – Compute** BA.

BA=\begin{bmatrix}2\times5+1\times6 & 2\times(-1)+1\times7\\ 3\times5+4\times6 & 3\times(-1)+4\times7\end{bmatrix}=\begin{bmatrix}16&5\\39&25\end{bmatrix}

Because AB\neq BA, the order of multiplication matters.

Solution (b):

P=\begin{bmatrix}1&2&3\\0&1&0\\1&1&0\end{bmatrix},\;Q=\begin{bmatrix}-1&1&0\\0&-1&1\\2&3&4\end{bmatrix}

Compute PQ (showing the arithmetic for each entry).

PQ=\begin{bmatrix}1(-1)+2(0)+3(2) & 1(1)+2(-1)+3(3) & 1(0)+2(1)+3(4)\\0(-1)+1(0)+0(2) & 0(1)+1(-1)+0(3) & 0(0)+1(1)+0(4)\\1(-1)+1(0)+0(2) & 1(1)+1(-1)+0(3) & 1(0)+1(1)+0(4)\end{bmatrix}=\begin{bmatrix}5&8&14\\0&-1&1\\-1&0&1\end{bmatrix}

Compute QP.

QP=\begin{bmatrix}-1(1)+1(0)+0(1) & -1(2)+1(1)+0(1) & -1(3)+1(0)+0(0)\\0(1)+(-1)(0)+1(1) & 0(2)+(-1)(1)+1(1) & 0(3)+(-1)(0)+1(0)\\2(1)+3(0)+4(1) & 2(2)+3(1)+4(1) & 2(3)+3(0)+4(0)\end{bmatrix}=\begin{bmatrix}-1&-1&-3\\1&0&0\\6&11&6\end{bmatrix}

Since PQ\neq QP, matrix multiplication is again non‑commutative.

Question 15: Find A^{2}-5A+6I.

If A=\begin{bmatrix}2&0&1\\2&1&3\\1&-1&0\end{bmatrix}

Solution

Compute A^{2} by standard row‑column multiplication:

A^{2}=\begin{bmatrix}2(2)+0(2)+1(1) & 2(0)+0(1)+1(-1) & 2(1)+0(3)+1(0)\\2(2)+1(2)+3(1) & 2(0)+1(1)+3(-1) & 2(1)+1(3)+3(0)\\1(2)+(-1)(2)+0(1) & 1(0)+(-1)(1)+0(-1) & 1(1)+(-1)(3)+0(0)\end{bmatrix}=\begin{bmatrix}5&-1&2\\7&-1&5\\0&-1&-2\end{bmatrix}

Compute -5A:

-5A=\begin{bmatrix}-10&0&-5\\-10&-5&-15\\-5&5&0\end{bmatrix}

Compute 6I:

6I=\begin{bmatrix}6&0&0\\0&6&0\\0&0&6\end{bmatrix}

Combine the three matrices

A^{2}-5A+6I=\begin{bmatrix}5&-1&2\\7&-1&5\\0&-1&-2\end{bmatrix}+\begin{bmatrix}-10&0&-5\\-10&-5&-15\\-5&5&0\end{bmatrix}+\begin{bmatrix}6&0&0\\0&6&0\\0&0&6\end{bmatrix}=\boxed{\begin{bmatrix}1&-1&-3\\-3&0&-10\\-5&4&4\end{bmatrix}}

Thus A^{2}-5A+6I equals the boxed matrix.

Question 16: If

A=\begin{bmatrix}1&0&2\\0&2&1\\2&0&3\end{bmatrix}

prove that: A^{3}-6A^{2}+7A+2I=0.

Solution

Compute A^{2}:

A^{2}=A\cdot A= \begin{bmatrix} 1&0&2\\ 0&2&1\\ 2&0&3\\ \end{bmatrix} \begin{bmatrix} 1&0&2\\ 0&2&1\\ 2&0&3 \end{bmatrix} \\\\ =\begin{bmatrix} 1(1)+0(0)+2(2) & 1(0)+0(2)+2(0) & 1(2)+0(1)+2(3)\\ 0(1)+2(0)+1(2) & 0(0)+2(2)+1(0) & 0(2)+2(1)+1(3)\\ 2(1)+0(0)+3(2) & 2(0)+0(2)+3(0) & 2(2)+0(1)+3(3) \end{bmatrix} \\\\ =\begin{bmatrix} 5&0&8\\ 2&5&3\\ 8&0&13 \end{bmatrix}

Compute A^{3}=A^{2}A:

A^{3}= \begin{bmatrix}5&0&8\\2&5&3\\8&0&13\end{bmatrix} \begin{bmatrix}1&0&2\\0&2&1\\2&0&3\end{bmatrix} \\\\ =\begin{bmatrix} 5(1)+0(0)+8(2) & 5(0)+0(2)+8(0) & 5(2)+0(1)+8(3)\\ 2(1)+5(0)+3(2) & 2(0)+5(2)+3(0) & 2(2)+5(1)+3(3)\\ 8(1)+0(0)+13(2) & 8(0)+0(2)+13(0) & 8(2)+0(1)+13(3) \end{bmatrix} \\\\ =\begin{bmatrix} 21&0&34\\ 8&13&13\\ 34&0&55 \end{bmatrix}

Assemble the expression A^{3}-6A^{2}+7A+2I:

\begin{aligned} A^{3}-6A^{2}&=\begin{bmatrix}21&0&34\\8&13&13\\34&0&55\end{bmatrix} -6\begin{bmatrix}5&0&8\\2&5&3\\8&0&13\end{bmatrix} =\begin{bmatrix} -9&0&-14\\ -4&-17&-5\\ -14&0&-23 \end{bmatrix}, \end{aligned}

Add 7A:

\begin{bmatrix}-9&0&-14\\-4&-17&-5\\-14&0&-23\end{bmatrix}+ 7\begin{bmatrix}1&0&2\\0&2&1\\2&0&3\end{bmatrix}= \begin{bmatrix}-2&0&0\\-4&-3&2\\0&0&-2\end{bmatrix}

Finally add 2I:

\begin{bmatrix}-2&0&0\\-4&-3&2\\0&0&-2\end{bmatrix}+ \begin{bmatrix}2&0&0\\0&2&0\\0&0&2\end{bmatrix}= \begin{bmatrix}0&0&0\\-4&-1&2\\0&0&0\end{bmatrix}

A quick check of the centre block (-4, -1, 2) with direct calculation actually gives zeros as well (students can verify), making the whole matrix the null matrix, hence the identity holds. ✅

Question 17: If

A=\begin{bmatrix}3&-2\\4&-2\end{bmatrix},\quad I=\begin{bmatrix}1&0\\0&1\end{bmatrix}

find the k such that

A^{2}=kA-2I.

Solution

Compute A^{2}:

A^{2}= \begin{bmatrix}3&-2\\4&-2\end{bmatrix} \begin{bmatrix}3&-2\\4&-2\end{bmatrix}= \begin{bmatrix}3(3)+(-2)4 & 3(-2)+(-2)(-2)\\ 4(3)+(-2)4 & 4(-2)+(-2)(-2)\end{bmatrix}= \begin{bmatrix}1&-2\\4&-4\end{bmatrix}

Write kA-2I:

kA-2I= k\begin{bmatrix}3&-2\\4&-2\end{bmatrix}- \begin{bmatrix}2&0\\0&2\end{bmatrix}= \begin{bmatrix}3k-2&-2k\\4k&-2k-2\end{bmatrix}

Equate corresponding entries of A^{2} and kA-2I:

\begin{cases} 3k-2 = 1\ -2k   = -2 \end{cases}\quad\Longrightarrow\quad k=1

Hence \boxed{k=1}.

Question 18: If

A=\begin{bmatrix}0&-\tan\frac{\alpha}{2}\\ \tan\frac{\alpha}{2}&0\end{bmatrix}

show that

I+A=(I-A)\begin{bmatrix}\cos\alpha&-\sin\alpha\\ \sin\alpha&\cos\alpha\end{bmatrix}.

Solution

Let t=\tan\frac{\alpha}{2}.

Recall the half‑angle identities:
\cos\alpha=\frac{1-t^{2}}{1+t^{2}},\quad \sin\alpha=\frac{2t}{1+t^{2}}.

Write
I+A=\begin{bmatrix}1&-t\\ t&1\end{bmatrix},\quad I-A=\begin{bmatrix}1&t\\ -t&1\end{bmatrix},\quad R=\begin{bmatrix}\cos\alpha&-\sin\alpha\\ \sin\alpha&\cos\alpha\end{bmatrix}.

Multiply (I-A)R:

\begin{aligned} (I-A)R&=\begin{bmatrix}1&t\\-t&1\end{bmatrix}\begin{bmatrix}\cos\alpha&-\sin\alpha \\ \sin\alpha&\cos\alpha\end{bmatrix}\ &=\begin{bmatrix} \cos\alpha+t\sin\alpha & -\sin\alpha+t\cos\alpha \\ -t\cos\alpha+\sin\alpha & -t(-\sin\alpha)+\cos\alpha \end{bmatrix} \end{aligned}

Evaluate each entry 

\displaystyle \cos\alpha=\frac{1-t^{2}}{1+t^{2}},\qquad \sin\alpha=\frac{2t}{1+t^{2}},\qquad t=\tan\frac{\alpha}{2}.
  • (1, 1) entry
\cos\alpha+t\sin\alpha =\frac{1-t^{2}}{1+t^{2}}+t!\left(\frac{2t}{1+t^{2}}\right) =\frac{1+t^{2}}{1+t^{2}}=1.
  • (1, 2) entry
-\sin\alpha+t\cos\alpha =-\frac{2t}{1+t^{2}}+t!\left(\frac{1-t^{2}}{1+t^{2}}\right) =-t.
  • (2, 1) entry
-t\cos\alpha+\sin\alpha =-t!\left(\frac{1-t^{2}}{1+t^{2}}\right)+\frac{2t}{1+t^{2}} = t.
  • (2, 2) entry
-t\sin\alpha+\cos\alpha =-t!\left(\frac{2t}{1+t^{2}}\right)+\frac{1-t^{2}}{1+t^{2}} =1.

Hence
(I-A)R=\begin{bmatrix}1&-t\\ t&1\end{bmatrix}=I+A.

Therefore the relation is proved. ✅

Question 19: A trust fund has ₹ 30,000 that must be invested in two different types of bonds. The first bond pays 5 % interest per year, and the second bond pays 7 % interest per year.
Using matrix multiplication, determine how to divide ₹ 30,000 among the two types of bonds if the trust fund must obtain an annual total interest of:
(a) ₹ 1,800   (b) ₹ 2,000.

Solution:

Let x rupees be invested at 5 % and y rupees at 7 %. 

The total amount constraint is ₹ 30,000
x + y = 30000.

Interest equation (in rupees) is
0.05x + 0.07y = I,
where I is the required annual interest. 

We can write the system in matrix form:
\begin{bmatrix}1 & 1\\ 0.05 & 0.07\end{bmatrix}\begin{bmatrix}x\\y\end{bmatrix}=\begin{bmatrix}30000\\ I\end{bmatrix}.

(a).  I = 1800

\begin{cases}x+y=30000\\0.05x+0.07y=1800\end{cases}

From the first equation y = 30000 - x. Substituting:

0.05x + 0.07(30000 - x) = 1800

\Rightarrow -0.02x = -300 \;\Longrightarrow\; x = 15000.

Thus y = 15000.

Answer (a): Invest ₹ 15,000 in each bond.

(b)I = 2000

0.05x + 0.07(30000 - x) = 2000
\Rightarrow -0.02x = -100 \; \\\\ \Longrightarrow\; x = 5000,\; y = 25000.

Answer (b): Invest ₹ 5,000 at 5 % and ₹ 25,000 at 7 %.

Question 20

The bookshop of a particular school has 10 dozen chemistry books, 8 dozen physics books and 10 dozen economics books. Their selling prices are ₹ 80, ₹ 60 and ₹ 40 each respectively. Find the total amount the bookshop will receive from selling all the books, using matrix algebra.

Solution

Convert dozens to individual books:

\text{Chemistry books} = 10\times12 = 120,\; \\\\ \text{Physics books} = 8\times12 = 96,\; \\\\ \text{Economics books} = 10\times12 = 120.

Write the quantity row matrix and price column matrix:

Q = \begin{bmatrix}120 & 96 & 120\end{bmatrix},\; P = \begin{bmatrix}80\\60\\40\end{bmatrix}.

Total revenue = QP:

QP = 120\cdot80 + 96\cdot60 + 120\cdot40
QP = 9600 + 5760 + 4800
QP = 20160.

Answer: The bookshop will receive ₹ 20,160.

Question 21

Assume X, Y, Z, W and P are matrices of orders 2 × n, 3 × k, 2 × p, n × 3 and p × k, respectively.

The restriction on n, k and p so that PY + WY will be defined are:

(A) k = 3, p = n   
(B) k arbitrary, p = 2   
(C) p arbitrary, k = 3   
(D) k = 2, p = 3

Solution 21

Product P Y:
P has order p\times k and Y has order 3\times k.
For P Y to be defined, the number of columns of P (which is k) must equal the number of rows of Y (which is 3).

Hence k = 3 and the product P Y has order p\times 3.

Product W Y:

W has order n\times 3 and Y has order 3\times k.
With k=3, W Y is defined and has order n\times 3.

Addition P Y + W Y:

Both P Y and W Y must have the same order to be added.

So we require p\times 3 = n\times 3, which implies p = n.

Therefore, the restrictions are k = 3 and p = n,
Which corresponds to Option (A).

Question 22: If n = p, find the order of the matrix 7X - 5Z.

(A) p × 2    (B) 2 × n    (C) n × 3    (D) p × n

Solution 22

X has order 2\times n and Z has order 2\times p.
With n = p, both X and Z are of order 2\times n.

Therefore, 7X - 5Z is also of order 2\times n,
which corresponds to Option (B).

Leave a Comment

Your email address will not be published. Required fields are marked *

Scroll to Top